Closed interval is covering compact

In summary, the question asks to prove directly that the closed interval [a, b] is covering compact, meaning that for any arbitrary open cover U of [a, b], there is a finite subcover. It is important to note that covering compact and compact are not necessarily the same thing. The usual proof for this statement is by contradiction, where one assumes that there is no finite subcover and then uses the least upper bound property to show that there must be a finite subcover. The proof involves splitting the interval into smaller subintervals and repeating the process until a single point is obtained. This proof can be applied to intervals of real numbers, but not necessarily to intervals of rational numbers.
  • #1
Scousergirl
16
0
The question asks to prove directly that the closed interval is covering compact

- U= an open covering of the closed set [a,b]
I started by taking C=the set of elements in the interval that finitely many members of U cover. Now I need to somehow use the least upper bound theorem to show that b is in C?
 
Physics news on Phys.org
  • #2
You want to show that for any arbitrary open cover of [a,b] there is a finite subcover.

Is covering compact the same thing as compact?
 
  • #3
Right so I assume U is my arbitrary open cover.

Covering compact implies compact but not vice versa right?
 
  • #4
What are your definitions of compact and covering compact?
 
  • #5
for any arbitrary open cover of [a,b] there is a finite subcover for covering compact but I the question isn't asking to use the definition of compact just covering compact.
 
  • #6
That is the definition of compact most people use. You've been given slightly non-standard notation. This is also called quasi-compact by some as well.

You're on the right lines. Show that the set of points you described (as having the finite covering property) is closed, then think again.
 
  • #7
The question asks to prove directly that the closed interval is covering compact

- U= an open covering of the closed set [a,b]
I started by taking C=the set of elements in the interval that finitely many members of U cover. Now I need to somehow use the least upper bound theorem to show that b is in C? ineed solution
 
  • #8
What makes you think that you can show that without showing, first, that there exist a finite subcover for the entire interval. That is NOT the usual proof for this statement.

The usual proof is by contradiction. Suppose there is NOT finite subcover for the interval. Now look at the two subintervals, [a, c], [c, b] where c is between a and b (for simplicity, you can choose it half way between- c= (a+b)/2. Since the entire interval cannot be covered by a finite collection of these open sets, at least one of the two subintervals cannot. Cut that interval into two pieces and repeat. Repeat until you get to a single point.

Of course, you have to be able to show that you do, in the limit, get a single point. For that you must specify that your interval is an interval or real numbers so that you can use the "least upper bound" and "greatest lower bound" properties. If you are thinking of [a, b] as an interval of rational numbers, the statement is not true.
 

Related to Closed interval is covering compact

1. What is a closed interval?

A closed interval is a set of real numbers that includes both its endpoints. It is represented as [a, b], where a and b are the endpoints and a ≤ b.

2. What is a compact set?

A compact set is a set of real numbers that is both closed (contains all its limit points) and bounded (can be contained within a finite interval).

3. What does it mean for a closed interval to cover a compact set?

It means that the closed interval contains all the points of the compact set. In other words, every point in the compact set is also included in the closed interval.

4. How is the statement "closed interval is covering compact" useful in mathematics?

This statement is useful in mathematical analysis as it helps prove the existence of solutions to certain problems, such as optimization and differential equations.

5. Can a closed interval cover a non-compact set?

No, a closed interval can only cover a compact set. If a set is not compact, it may not have a finite interval that contains all its points.

Similar threads

Replies
4
Views
2K
  • Calculus
Replies
3
Views
1K
  • Calculus and Beyond Homework Help
Replies
12
Views
1K
Replies
3
Views
958
  • Topology and Analysis
Replies
5
Views
314
  • Calculus
Replies
9
Views
2K
  • Topology and Analysis
Replies
32
Views
2K
Replies
3
Views
383
Replies
1
Views
1K
Replies
9
Views
954
Back
Top